You are on page 1of 10

1

Sample Question Paper 11


(Detailed Solutions)

Mathematics
Class 10th
1. According to Euclid’s division lemma, 5. Let two circles with same centre at O. And PR be the
153 = 85 ´ 1 + 68 chord which touches the inner circle.
Þ 85 = 68 ´ 1 + 17
and 68 = 17 ´ 4 + 0
\ HCF of 85 and 153 is 17. (1/2)
O
According to the question,
5 cm
17 = 85 m - 153 3 cm
P R
Þ 85 m = 170 Q

170
\ m= =2
85 (1/2)
Also, OQ ^ PR
2. Given quadratic equation is [Q tangent PR at point Q of an
2x 2 + x + 4 = 0 incircle is perpendicular to the radius
through the point of contact]
On comparing with ax 2 + bx + c = 0, we get
\ ÐOQP = 90°
a = 2, b = 1 and c = 4
PQ = QR …(i)
Now, discriminant, D = b 2 - 4ac
[Q perpendicular from the centre bisects the chord]
= (1)2 - 4 ´ 2 ´ 4
In DOQP, (OP )2 = ( PQ )2 + (OQ )2
= 1- 32 = -31 < 0
[by Pythagoras theorem]
which is not real.
Þ ( 5) = ( PQ ) + ( 3)2
2 2
Hence, there are no real roots for the given
[given, OP = 5 and OQ = 3 ] (1/2)
equation. [1]
Þ ( PQ )2 = 25 - 9 = 16
2
3. Given equation is x - 4x + 1 = 0 Þ PQ = 4 cm [taking positive square root]
On comparing the given equation with Þ PR = PQ + QR
ax 2 + bx + c = 0, we get (1/2) Þ PR = PQ + PQ [from Eq. (i)]
a = 1, b = -4 and c = 1 =4+4
2
\Discriminant, D = b - 4ac = 8 cm (1/2)
2
= ( -4) - 4(1)(1) 6. Since, the first five prime numbers are 2, 3, 5, 7
= 16 - 4 = 12 (1/2) and 11.
4. Given that, k + 9, 2 k - 1, 2 k + 7 are in AP. \ Sum of the numbers = 2 + 3 + 5 + 7 + 11 =28 (1/2)
\ a2 - a1 = a3 - a2 Here, n=5
where a1, a2 and a3 are the three consecutive terms 28
\ Mean = = 5.6
of an AP. (1/2) 5
Þ (2 k - 1) - ( k + 9) = (2 k + 7 ) - (2 k - 1) éQmean= sum of observations ù
Þ k - 10 = 8 êë total number of observations úû (1/2)
Þ k = 18 (1/2)
2
7. For the maximum number of columns, we have to 1
OB = BD = x cm
find the HCF of 616 and 32. 2
Since, 616 > 32, so we apply Euclid’s division lemma and ÐAOB = 90° (1/2)
to 616 and 32. In right angled D AOB, by Pythagoras theorem
We have, 616 = 32 ´ 19 + 8 (1/2) AB2 = OA 2 + OB2
Here, remainder 8 ¹ 0. So, we again apply Euclid’s Þ OB2 = AB2 - OA 2
division lemma to 32 and 8. Þ x 2 = (10)2 - (6 )2
We have, 32 = 8 ´ 4 + 0 (1/2) Þ x 2 = 100 - 36
Here, remainder is zero. So, HCF (616,32) = 8 Þ x 2 = 64 Þ x = 64 =8
Hence, the maximum number of columns is 8. (1) [taking positive square root] (1/2)
1 \ OB = 8 cm
8. Let a and be two zeroes of the given polynomial,
a Then, BD = 2 ´ OB = 2 ´ 8 = 16 cm
which are reciprocal of each other. Hence, the length of the second diagonal is 16 cm.
Now, product of zeroes, (1/2)
1 Constant term 6a 11. Given A DPQR such that
a´ = 2
Þ 1= 2
a Coefficient of x a +9 (1) DE || AQ and DF || AR.
To prove EF || QR
Þ a 2 + 9 = 6 a Þ a 2 - 6 a + 9 = 0 Þ ( a - 3)2 = 0
Proof In DPQA,
[Q x 2 + y 2 - 2 xy = ( x - y )2]
DE || AQ [given]
\ a=3 (1)
PE PD
\ = …(i) (1/2)
9. Given, points P ( 4, 3) and Q ( x , 5) are on the circle EQ DA
with centre O (2, 3). [by basic proportionality theorem]
Also, in D PRA,
O (2, 3)
P (4, 3) Q (x, 5) DF || AR [given]
PD PF
\ = …(ii) (1/2)
DA FR
\ OP = OQ (1/2)
[by basic proportionality theorem]
Þ ( 4 - 2 )2 + ( 3 - 3)2 = ( x - 2 )2 + ( 5 - 3)2 From Eqs. (i) and (ii),
PE PF
[by distance formula] (1/2) = (1/2)
EQ FR
On squaring both sides, we get
i.e. EF divides PQ and PR in the same ratio.
( 4 - 2 )2 + ( 3 - 3)2 = ( x - 2 )2 + ( 5 - 3)2
Þ (2 )2 = ( x - 2 )2 + (2 )2 Hence, by converse of basic proportionality
Þ ( x - 2 ) + (2 ) - (2 )2 = 0 Þ ( x - 2 )2 = 0
2 2 theorem, EF || QR. Hence proved.(1/2)
Þ x -2= 0 12. Let O be the centre of concentric circles and PQ be
\ x =2 the chord of bigger circle, which touches the smaller
Hence, the value of x is 2. (1) circle at M.

10. Let ABCD be the given rhombus D C


whose diagonals intersect at O.
Ox
Then, AB = 10 cm
cm

c m O
5c
6

m
Let AC = 12 cm 5c
m
3 cm

A 10 cm B
and BD = 2 x cm . P Q
M
We know that the diagonals of a rhombus bisect
each other at right angles.
Then, PQ = 2PM = 2QM
1 [Q chord PQ is a tangent to the smaller circle,
\ OA = AC = 6 cm (1/2)
2 therefore OM ^ PQ and hence OM bisect PQ] (1)
3
Now, in right angled DPMO at M, and 7 x + 1 - 5y + 1 = 218 Þ 7 × 7 x - 5 × 5y = 218
= 90° Let 7 x = p and 5y = q (1/2)
Use Pythagoras theorem, we get Then, given equations become
2 2
PM = PO - OM = 25 - 9 = 16 = 4 cm p + q = 74 ...(i)
and 7 p - 5 q = 218 ...(ii) (1)
Thus, PQ = 2 PM = 2 ´ 4 = 8 cm
Hence, the length of the chord of the larger circle On putting p = 74 - q from Eq. (i) in Eq. (ii), we get
which touches the smaller circle is 8 cm. (1) 7(74 - q ) - 5q = 218
13. Let a be any positive integer. On dividing a by 3, let q Þ - 12q = 218 - 518 = - 300
be the quotient and r be the remainder. By Euclid’s \ q = 25 (1/2)
and division Lemma, we have a = 3q + r, where
0 £ r < 2 i.e. r = 0,1, 2 On substituting the value of q in Eq. (i), we get
(i) When r = 0, a = 3q p + 25 = 74 Þ p = 49
Thus the three consecutive positive integers are Thus, we get p = 49 and q = 25
n = 3q , n + 1 = 3q + 1 and n + 2 = 3q + 2. \ 7 x = 49 and 5y = 25
Out of these, 3q is divisible by 3, 3q + 1 is not Þ 7 x = 7 2 and 5y = 52
divisible by 3 and 3q + 2 is also not divisible by 3.
(1) \ x = 2 and y = 2 [Q m a = m b Þ a = b]
(ii) When r = 1, a = 3q + 1 Hence, the solution of the given system of
Thus the three consecutive positive integer are equations is x = 2 and y = 2. Hence proved. (1)
n = 3q + 1, n + 1 = 3q + 2 and n + 2 = 3q + 3.
15. (i) Given pair of linear equations is
Out of these 3q + 1is not divisible by 3, 3q + 2 is
5x - 4 y + 8 = 0
not divisible by 3 and 3q + 3 = 3(q + 1) is divisible
by 3. (1) and 7x + 6 y - 9 = 0

(iii) When r = 2, a = 3q + 2 On comparing with standard equations

Thus the three consecutive positive integers are a1x + b1y + c 1 = 0


n = 3q + 2, n + 1 = 3q + 3 and n + 2 = 3q + 4 and a2x + b2 y + c 2 = 0, we get
Out of these, 3q + 2 is not divisible by 3, a1 = 5, b1 = - 4, c 1 = 8
3q + 3 = 3(q + 1) is divisible by 3 and and a2 = 7, b2 = 6, c 2 = - 9 (1/2)
3q + 4 = 3(q + 1) + 1 is not divisible by 3. a1 5 b1 4 2
Now, = , =- =-
Hence, in all cases, we see that one of the three a2 7 b2 6 3
consecutive positive integers must be divisible
c1 8
by 3. Hence proved. (1) and =-
c2 9
Or
a1 b1
Since, the remainders are 4,5 and 6, respectively. \ ¹
a2 b2 (1)
So, we have to find the HCF of 445 - 4, 572 - 5
and 699 - 6, i.e. 441, 567 and 693. (1/2) Hence, the lines representing the given
equations are intersecting at a unique point.
For the HCF of 441, 567 and 693, we have
(ii) Given pair of linear equations is
441 = 3 ´ 3 ´ 7 ´ 7 = 32 ´ 7 2 (1/2)
9x + 3 y + 12 = 0
567 = 3 ´ 3 ´ 3 ´ 3 ´ 7 = 34 ´ 7 (1/2)
and 693 = 3 ´ 3 ´ 7 ´ 11 = 32 ´ 7 ´ 11 (1/2)
and 18x + 6 y + 24 = 0
\ HCF of 441, 567 and 693 = 32 ´ 7 = 63 On comparing with standard equations
Hence, required number = 63 (1) a1x + b1y + c 1 = 0
14. The given equations here are not the linear and a2x + b2 y + c 2 = 0, we get (1/2)
equations. So, first of all convert them into linear a1 = 9, b1 = 3, c 1 = 12
equations by putting 7 x = p and 5 y = q . Then, solve and a2 = 18, b2 = 6, c 2 = 24
by suitable method and put the values of p and q. To a1 9 1 b 3 1
find the values of x and y, by using the property Now, = = , 1= =
a2 18 2 b2 6 2
m a = m b , then a = b.
Given, 7 x + 5y = 74
4
c 1 12 1 i.e. y + x = 27 …(ii) (1)
and = =
c 2 24 2
On substituting the value of x from Eqs. (i) to (ii),
a1 b1 c 1 we get y + 8 y = 27 Þ 9 y = 27 Þ y = 3
\ = =
a2 b2 c 2 On putting y = 3 in Eq. (ii), we get x = 8( 3) = 24
Hence, the lines representing the given equation Hence the number of lines in blue colour wheel is
are coincident and it has infinitely many points. 24. (1)
(1)
Or 17. Let 1 woman alone can finish the work in x days.
We have 1
\ 1 woman’s one day’s work =
2x - y = 3 …(i) x
and 3x + 2 y = 8 …(ii) Let 1 man alone can finish the work in y days.
Let us draw the graphs of equations (i) and (ii). 1
For this, we find two solutions of each of the \ 1 man’s one day’s work =
y
equations.
Table for 2 x - y = 3 is Since, 2 women and 5 men can together finish an
embroidery work in 4 days.
x 0 2
y -3 1
\ 2 women’s one day’s work
1
Table for 3x + 2 y = 8 is + 5 men’s one day’s work =
4
x 2 0 2 5 1
Þ + = ... (i)
y 1 4 x y 4 (1)
(1)
Again, 3 women’s one day’s work + 6 men’s one
Plot the points A( 0, - 3), B(2, 1) and C( 0, 4) on graph 1
day’s work =
paper and join the points to form the lines AB and 3
BC as shown in the figure below. 3 6 1
Y Þ + = ... (ii)
x y 3
C (0, 4)
4 1 1
Let = u and = v , then Eqs. (i) and (ii) become
x y
3x

3
+2
y=

2 1
2u + 5v = ... (iii)
8

1 B (2, 1) 4
1
X′
–4 –3 –2 –1 O 1 2 3 4 5 6
X and 3u + 6 v = ... (iv)
3
–1
3
y=

On multiplying Eq. (iii) by 3 and Eq. (iv) by 2


2x–

–2
and then subtracting Eq. (iv) from Eq. (iii), we get
–3 A (0, –3)
3 2
–4 (6u + 15v ) - (6u + 12 v ) = -
4 3
Y′ (1) 9- 8 1 1
Þ 3v = Þ v= = (1)
The two lines of euqations (i) and (ii) intersect at the 12 3 ´ 12 36
point B(2, 1.
) So, x = 2, y = 1 is the required solution 1
of the pair of equations. (1) Put v = in Eq. (iii), we get
36
16. Let the number of lines in blue colour wheel be x and 1 1 1 5
2u + 5 ´ = Þ 2u = -
the number of colours in the flag be y. 36 4 4 36
It is given that 9- 5 4 1
Þ 2u = Þ 2u = Þ u=
Number of lines in blue colour wheel 36 36 18
1 1
= 8 times the number of colours in the flag \ u= and v =
18 36
i.e. x = 8y …(i) (1)
1 1 1 1
Þ = and =
and, the sum of number of colours in the flag and x 18 y 36
number of lines in the wheel of the flag is 27.
Þ x = 18 and y = 36 (1)
5
Or Þ ÐOBD = 30 ° (1)
According to the question, Now, in DOBD,
a + b = 15 …(i) BD é base ù
1 1 3 cos 30° =
OB êQcosq = hypotenuse ú
and + = ë û
a b 10
3
1 1 3 Þ BD = OB cos 30 ° = 4 ´ = 2 3 cm
Þ + = [Q a + b = 15 Þ b = 15 - a] 2
a 15 - a 10
(1) é 3ù
Þ
15 - a + a
=
3 êQOB = r = 4cm, given and cos30 ° = 2 ú
ë û
a (15 - a ) 10
Þ BC = 2 ´ 2 3 = 4 3 cm (1/2)
Þ 3a (15 - a ) = 150
éQ D ODB @ D ODC ù
Þ 45a - 3a 2 - 150 = 0
êë\ BD = DC Þ BC = 2 BDúû
Þ -3a 2 + 45a - 150 = 0
Þ a 2 - 15a + 50 = 0 Clearly, area of shaded region
[dividing both sides by -3] 1
= (Area of circle – Area of an equilateral DABC)
Þ a 2 - 5a - 10a + 50 = 0 [by factorisation] 3
(1/2)
Þ a( a - 5) - 10 ( a - 5) = 0 Þ ( a - 10) ( a - 5) = 0 (1)
\ a = 10 or 5 1é 3 ù
= p ( 4) 2 - ( BC )2 ú [Q r = 4 cm]
If a = 10, then from Eq. (i), b = 15 - 10 = 5 3 êë 4 û
If a = 5, then from Eq. (i), b = 15 - 5 = 10 1 é 22 ´ 16 3 ( 4 3 )2 ù
= - [Q BC = 4 3 cm]
Hence, the required numbers are 10 and 5. (1) 3 êë 7 4 ú
û
18. Given, DABC is an equilateral triangle 1 æ 22 ´ 16 3 ´ 48 ö
= ç - ÷
\ ÐA = 60° Þ ÐBOC = 120° 3è 7 4 ø
[Q angle subtended by an arc at the centre is 1 æ 88 ´ 4
= ç - 3 ´ 12 ö÷
double the angle subtended by the same arc at 3è 7 ø
remaining part of the circle.] 4 88 4 88 - 36.33 ö
= æç - 3 3 ö÷ = æç ÷ [Q 3 = 1.73]
Now, draw OD ^ BC. Then, DODB @ DODC è
3 7 ø 3è 7 ø
[by RHS criterion] 4
= ´ 7.38 = 9.84 cm 2 (1)
3

19. Let a be the first term and d be the common


difference of the given AP. Then, the sums of first
m
m and n terms are given by S m = [2 a + ( m - 1) d ]
2
n
and S n = [2 a + ( n - 1) d ], respectively
2 (1/2)
Sm m2
\ ÐBOD = ÐCOD [by CPCT] …(i) Since, = 2
Sn n
ÐBOC = ÐBOD + ÐCOD m
[2 a + ( m - 1) d ]
Þ 120° = ÐBOD + ÐBOD 2 m2
\ = 2
[from Eq. (i) and ÐBOC = 120 °] n n
[2 a + ( n - 1) d ]
Þ 2 ÐBOD = 120 ° Þ ÐBOD = 60 ° 2
2 a + ( m - 1) d m
Applying angle sum property in DODB, we get Þ =
2 a + ( n - 1) d n (1)
ÐOBD + ÐODB + ÐBOD = 180 °
Þ [2 a + ( m - 1) d ] n = [2 a + ( n - 1) d ] m
Þ ÐOBD + 90 ° + 60 ° = 180 °
Þ 2 a ( n - m ) = d [( n - 1) m - ( m - 1) n]
[Q ÐODB = 90 ° and ÐBOD = 60 °]
6
Y
Þ 2a ( n - m ) = d ( n - m ) Þ d = 2 a (1/2)
a a + ( m - 1) d
Now, required ratio = m = 100 (130, 100)
an a + ( n - 1)d 90
(120, 90)

Cumulative frequency
80
a + ( m - 1) 2 a 2 m - 1 70 (110, 78)
= = 60 (100, 64)
a + ( n - 1) 2 a 2n - 1 50
40 (90, 46)
or am : an = (2 m - 1) : (2 n - 1) Hence proved. 30 (80, 30)
(1) 20
(70, 18)
2 cm 10
20. For cylindrical part, X′
(60, 8)
X
0 60 70 80 90 100 110 120 130 140
2
r = = 1cm, 8 cm Upper limits of class interval (Weight in gm)
2 Y′ (1)
and h = 8 cm …(i) 22. Calculation of mean
For spherical part, Daily Savings Number of Class-mark
8.5 cm fi x i
8.5 17 (in `) children ( fi ) ( xi )
R= = cm …(ii) (1)
2 4 1- 3 7 2 14
Clearly, volume of glass vessel 3-5 6 4 24
= Volume of cylindrical part 5-7 x 6 6x
+ Volume of the spherical part 7-9 13 8 104
4 4
= pr 2h + pR 3 = p æç r 2h + R 3 ö÷ (1) 9 - 11 y 10 10y
3 è 3 ø
11 - 13 5 12 60
4 17 17 17 ö 13 - 15 4 14 56
. æç1 ´ 1 ´ 8 + ´
= 314 ´ ´ ÷
è 3 4 4 4ø Total n = Sfi = 35 Sfi xi = 258
+ x+ y + 6 x + 10 y
[from Eqs. (i) and (ii)]
314 æ 4913 ö 314 æ 384 + 4913 ö (1)
= ç8 + ÷= ç ÷
100 è 48 ø 100 è 48 ø
From the table,
314 5297 1663258 35 + x + y = 64 [given, Sfi = 64]
= ´ =
100 48 4800 Þ x + y = 64 - 35
= 346.51 cm 3 Þ x + y = 29
Hence, she is not correct. (1)
Þ y = 29 - x …(i)
21. The cumulative frequency table is given below Sf x
Using the formula, Mean = i i
Cumulative
Sfi (1)
Class Frequency
Frequency
258 + 6 x + 10 y
50-60 8 8 Þ 8= [given, mean = 8]
64
60-70 10 18 Þ 512 = 258 + 6 x + 10(29 - x ) [Using Eq. (i)]
70-80 12 30 Þ 512 = 258 + 6 x + 290 - 10x
80-90 16 46 Þ 4x = 548 - 512
90-100 18 64 Þ 4x = 36 Þ x = 9 …(ii)
100-110 14 78 From Eqs. (i) and (ii), we get
110-120 12 90 y = 29 - 9 = 20
120-130 10 100 Hence, the missing frequencies are x = 9 and
(1) y = 20. (1)
Now, mark the upper limits on X -axis and cumulative Or
frequencies on Y-axis, by choosing convenient scale.
Let the frequency of class 20-30 be f1 and that of class
Now, plot the points (60, 8), (70, 18), ( 80, 30), ( 90, 46 ), 40-50 be f2.
(100, 64), (110, 78), (120, 90), (130, 100) on a graph
paper and join all these points by a free hand curve. Since, the sum of all frequency is 170.
(1) The cumulative frequency table for given distribution is
7
Class interval Frequency Cumulative frequency Hence the points A( 3, - 1), B( 5, - 1) and C( 3, - 3) are
the vertices of a right angled isosceles triangle. (1)
0-10 10 10
1
10-20 20 30 (ii) Now, area of DABC = ´ base ´ altitude
2
20-30 f1 30 + f 1 1 1
= ´ AB ´ AC = ´ 2 ´ 2 = 2 sq units
30-40 40 70 + f 1 2 2 (1)
40-50 f2 70 + f 1 + f 2 Or
50-60 25 95 + f 1 + f 2 Let us suppose that the given points ( a, a 2 ), ( b, b 2 )
60-70 15 110 + f 1 + f 2 and (c, c 2 ) are collinear.

Here, median = 35 (1) Then, condition for collinearity,


N 1
So, the median class is 30-40. Also, N = 170 Þ = 85 | x 1( y2 - y3 ) + x 2( y3 - y1) + x 3( y1 - y2 )| = 0
2 2
\ l = 30, f = 40, cf = 30 + f 1 and h = 10 (1/2) 1
\ [|a ( b 2 - c 2 ) + b (c 2 - a 2 ) + c ( a 2 - b 2 )|] = 0
ì N - cf ü 2 (1)
ï ï
Now, median = l + í 2 ý´h Þ |ab 2 - ac 2 + bc 2 - a 2b + a 2c - cb 2| = 0
ï f ï
î þ Þ - a 2 ( b - c ) + a ( b 2 - c 2 ) - bc ( b - c ) = 0 (1)
Þ ( b - c ) { - a 2 + a ( b + c ) - bc} = 0
85 - ( 30 + f1)ü
Þ 35 = 30 + ìí ý ´ 10 Þ ( b - c ) ( - a 2 + ab + ac - bc ) = 0
î 40 þ (1/2)
Þ ( b - c ) { - a ( a - b ) + c ( a - b )} = 0
Þ 35 ´ 4 = 120 + ( 55 - f1) Þ ( b - c ) ( a - b ) (c - a ) = 0 (1)
Þ 140 = 175 - f1 Þ b=c or a = b or c = a
Þ f1 = 35 …(i) But it is given that, a ¹ b ¹ c
Also, 110 + f1 + f2 = 170 So, our supposition is wrong.
[Qsum of all frequencies = 170, given] Hence, the points ( a, a 2 ), ( b, b 2 ) and (c, c 2 ) can
Þ f 1 + f 2 = 60 never be collinear. Hence proved. (1)
Þ 35 + f 2 = 60 [from Eq. (i)] 24. Since, XY|| X ¢ Y ¢ and CD is the transversal, therefore,
\ f 2 = 60 - 35 = 25
ÐACP + ÐPDB = 180° …(i)
Hence, the missing frequency of the class 20-30 is 35
éQsum of the interior angles on the ù
and the class 40-50 is 25. (1)
ëê same side of a transversal is180° ûú
23. (i) Given points are A( 3, - 1), B( 5, - 1) and C( 3, - 3)
Join OC and OD.
Using the distance formula,
X A C Y
AB = ( 5 - 3)2 + ( -1+ 1)2 = 4 + 0 = 2

[Q distance = ( x 2 - x 1)2 + ( y2 - y1)2 ]


O P
BC = ( 3 - 5)2 + ( -3 + 1)2 = 4 + 4 = 2 2

and AC = ( 3 - 3)2 + ( -3 + 1)2 = 0 + 4 = 2 (1)


X′ B D Y′
Here, we see that AB = AC [Q Each = 2 units]
Hence, DABC is an isosceles triangle. In DAOC and DPOC, we have
2 2 2 2
Now, AB + AC = (2 ) + (2 ) = 4 + 4 = 8 CA = CP (1)
and 2
BC = (2 2 ) = 8 2 [Q tangents drawn from an external
points are equal in length]
\ AB2 + AC 2 = BC 2
CO = CO [common]
Þ ÐBAC = 90°
OA = OP [ radii of the same circle]
[by converse of Pythagoras theorem] (1)
\ DAOC ~ DPOC [by SSS similarity criterion]
Þ DABC is qight angled triangle.
Þ ÐACO = ÐPCO …(ii) [by CPCT](1)
8
Similarly, we can prove A
ÐPDO = ÐBDO …(iii)
13 5
Now, Eq. (i) can be written as
( ÐACO + ÐPCO ) + ( ÐPDO + ÐBDO ) = 180°
Þ ( ÐPCO + ÐPCO ) + ( ÐPDO + ÐPDO ) = 180° B C
12
[using Eqs. (ii) and (iii)] By Pythagoras theorem, we have
Þ 2 ÐPCO + 2 ÐPDO = 180° AB2 = BC 2 + AC 2 Þ AB2 = 12 2 + 52 = 169
Þ 2( ÐPCO + ÐPDO ) = 180° Þ AB = 169 = 13
Þ ÐPCO + ÐPDO = 90° …(iv) (1) [taking positive square root]
In DCOD, we have AC 5 é perpendicular ù
\ sin B = = , êQsinq = hypotenuse ú
ÐPCO + ÐPDO + ÐCOD = 180° AB 13 ë û
[by angle sum property of a triangle] AC 5 éQ tanq = perpendicular ù
tan B = =
Þ ( ÐPCO + ÐPDO ) + ÐCOD = 180° BC 12 êë base úû

90° + ÐCOD = 180° [using Eq. (iv)] AB 13 éQsec q = hypotenuse ù


Þ and sec B = =
BC 12 êë base úû
Þ ÐCOD = 180°-90° = 90°
(1)
Þ Angle in a semi-circle is 90°, i.e. when a circle 2 2
Now, LHS = tan B - sin B = (tan B) - (sin B) 2 2
with P as centre and radius OP will pass through C
5 2 5 2 25 25
and D. (1) = æç ö÷ - æç ö÷ = -
è 12 ø è 13 ø 144 169
25. Steps of construction.
1 1 ö æ 169 - 144 ö
(i) On taking a point O as centre, draw a circle of = 25 æç - ÷ = 25 ç ÷
è 144 169 ø è 144 ´ 169 ø
radius 3 cm.
25 ´ 25 52 ´ 52 54
(ii) On taking a point P, 7 cm away from its centre O = = 2 = …(i)
144 ´ 169 12 ´ 132 12 2 ´ 132 (1½)
such that OP = 7 cm.
(iii) Bisect OP and let M be the mid-point of OP. (1) RHS = sin4 B sec 2 B = (sin B)4 (sec B)2
5 4 13 2 54 ´ 132 54
= æç ö÷ ´ æç ö÷ = 4 =
è 13 ø è 12 ø 13 ´ 12 2 132 ´ 12 2
T1 …(ii) (1)
\ LHS = RHS [from Eqs. (i) and (ii)]
Hence proved. (1/2)
3 cm M
O P
7 cm 27. We have a = tanq - cot q
On squaring both sides, we get
T2 a 2 = (tanq - cot q )2
= tan2 q + cot 2 q - 2 tanq cot q
(1) = tan2 q + cot 2 q - 2 (1)
(iv) Draw a circle with M as centre and OM as radius 2 2
\ a + 4 = tan q + cot q - 2 + 4 2
to intersect the circle at T1 and T2. (1)
= (1+ tan2 q ) + (1+ cot 2 q )
Then, PT1 and PT2 are the required tangents.
= sec 2 q + cos ec 2q
(v) Length of the each tangent 1 1
= +
PT1 = PT2 = 6.32 cm (approx.) (1) cos 2 q sin2 q

26. We have, cot B=


Base
=
12 sin2 q + cos 2 q 1
= =
Perpendicular 5 cos 2 q sin2 q cos 2 q sin2 q (1)
So, we draw a right angled DACB, right angled at C and b = cos q + sinq
such that base = BC = 12 units
On squaring both sides, we get
and perpendicular = AC = 5 units
b 2 = (sinq + cos q )2 = sin2 q + cos 2 q + 2 sinq cos q
9
b 2 = 1+ 2 sinq × cos q Þ b 2 - 1 = 2 sinq × cos q and ÐADB = ÐXAD = 60°
2 2
[Qsin q + cos q = 1] In right angled DABC, we have
2 2
b = 1+ 2 sin q × cos q Þ b - 1 = 2 sinq × cos q AB
= tan30°
Þ ( b 2 - 1)2 = (2 sinq cos q )2 = 4sin2 q cos 2 q (1) BC
AB AB
1 Þ BC = = = AB 3 ... (i)
\ ( a 2 + 4)( b 2 - 1)2 = ´ 4sin2 q cos 2 q tan 30° 1/ 3
cos 2 q sin2 q
Þ ( a 2 + 4)( b 2 - 1)2 = 4 Hence proved. (1) éQ tan30° = 1 ù (1)
ëê 3 úû
Or
In right angled DABD, we have
cosec 65° - tan2 25°
2
AB
We have, = tan60°
sin2 17° + sin2 73° BD
1
+ (tan 10° tan 30° tan 80°) Þ AB = BD tan60° = BD 3 ... (ii)
3 [Q tan60° = 3 ] (1)
cosec 2 ( 90 ° - 25 ° ) - tan2 25 °
= (1) \ From Eqs. (i) and (ii), we get
sin2 17 ° + sin2 ( 90 ° - 17 ° )
BC = AB 3 = BD 3 ´ 3 = 3BD
1
+ [tan10 ° tan 30 ° tan( 90 ° – 10 ° )] Þ BD + CD = 3BD [Q BC = BD + CD]
3
Þ CD = 2 BD
sec 2 25 ° - tan2 25 °
= Let the car takes x seconds to reach from D to B
sin2 17 ° + cos 2 17 °
which is the foot of the tower.
1
+ [tan 10 ° tan 30 ° cot 10 °] (1) CD
3 Speed of car travelling from C to D =
6
éQcosec ( 90 ° – q ) = sec q ù
éQspeed = distance ù
ê sin( 90 ° – q ) = cos q ú êë time úû
ê and tan ( 90 ° – q ) = cot q ú
ë û
DB
Speed of car travelling from D to B =
1 1 æ 1 ö x
= + ç tan10 ° tan 30 °× ÷
1 3è tan10 ° ø (1) Since the car is approaching with a uniform speed.
é (1)
êQsec 2 q - tan2 q = 1, sin2 q + cos 2 q = 1 DB CD 2 BD
\ = = [QCD = 2 BD, proved above]
ê x 6 6
ë ù
1 1 1
and cot q = ú Þ = Þx =3
tanq ú x 3
û
Hence, the further time taken by the car to reach the
1 1 éQ tan30 ° = 1 ù foot of the tower = 3 s (1)
= 1+ ×
3 3 êë 3 úû
Or
1 4
= 1+ = (i) Let B be the point on the ground, which is 60 m
3 3 (1) away from the foot of tower AM.
28. Let AB be the tower and C and D are two positions of C
the car. xm
X
A M
30°
60°
hm
45° 30°
B
A 60 m

Let h m be the height of the tower.


30° 60° In right angled DMAB,
C D B h é Perpendicular ù
tan 30° = Q tanq =
Let AX be the horizontal line through A. 60 ëê Base úû
We have, ÐACB = ÐXAC = 30°
10
1 h éQ tan30° = 1 ù (1) (b) Since, both persons are standing on the
Þ =
3 60 ëê 3 úû circular line. Therefore, the distances of both
60 60 3 60 3 persons are same. (1)
\ h= = ´ =
3 3 3 3 (iii) By walking, person shows that exercise is good
= 20 3 = 34.64 m [Q 3 = 1732
. ] for health. (1/2)
30. Q Total cards bearing numbers from 1 to 17 = 17
Thus, the height of the tower is 34.64 m (approx).
(1½) (i) Let E1 denote the event that card bears an odd
(ii) Let the depth of the water tank CM be x m. number. i.e. = {1, 3, 5, 7, 9, 11, 13, 15, 17}
In right angled DCAB, \ Number of outcomes favourable to E1 = 9
AC h + x
\ tan 45° = = Now,
AB 60 Number of outcomes favourable to E1
P( E1) =
[Q AC = AM + MC = h + x and AB = 60 m] Total number of outcomes
h+ x 9
Þ =1 [Q tan 45 ° = 1] =
60 17 (1)
Þ h + x = 60 (ii) Let E2 denote the event that the card bears a
\ x = 60 - h = 60 - 3464
. = 25.36 m prime number. i.e. = {2, 3, 5, 7, 11, 13, 17}
Hence, the depth of the water tank is \ Number of outcomes favourable to E2 = 7
25.36 m (approx). (1½) Now,
Number of outcomes favourable to E2
29. Given, radius of circular park, r1 = 10 m P( E2 ) =
Total number of outcomes
and radius of circular line, r2 = 11m
7
=
17 (1)
11 m (iii) Let E3 be the event that the card bears a number
A B that is divisible by 3. i.e. = { 3, 6, 9, 12, 15}
10 m
\ Number of outcomes favourable to E3 = 5
\ P( E3 )
Number of outcomes favourable to E3
(i) Clearly, perimeter of circular line = 2 p r2 =
Total number of outcomes
= 2 ´ 3.14 ´ 11 = 69.08 m (1/2) 5
= (1)
As the person takes four rounds, therefore the 17
total distance covered by him
(iv) Let E4 denote the event that the card bear a
= 4 ´ Perimeter of one circular line
number divisible by 2 and 3 both. i.e. = {6, 12}
= 4 ´ 69.08 = 276.32 m (1)
\ Number of outcomes favourable to E4 = 2
(ii) (a) Since, two persons are standing on the
opposite sides of a circular line, so the Now, P( E4 )
maximum distance between A and B will be Number of outcomes favourable to E4 2
= =
equal to the diameter of the circular line. Total number of outcomes 17
i.e. 2 r2 = 2 ´ 11 = 22 m (1) (1)

You might also like